Oscillating a spring

A block of mass M M is connected to a wall by two springs of respective spring constants k 1 k_1 and k 2 k_2 . The block shows simple harmonic oscillation with amplitude A A .

Find the amplitude of oscillation of point P P where the two springs are connected.

A A k 1 A k 2 \frac{k_1 A}{k_2} k 2 A k 1 \frac{k_2 A}{k_1} k 1 A k 1 + k 2 \frac{k_1 A}{k_1+k_2} k 2 A k 1 + k 2 \frac{k_2 A}{k_1+k_2}

This section requires Javascript.
You are seeing this because something didn't load right. We suggest you, (a) try refreshing the page, (b) enabling javascript if it is disabled on your browser and, finally, (c) loading the non-javascript version of this page . We're sorry about the hassle.

5 solutions

Sardor Yakupov
Jul 21, 2017

Relevant wiki: Hooke's Law

At the point P there are 2 Forces (at t=0): k 1 A 1 { k }_{ 1 }{ A }_{ 1 } and k 2 A 2 = k 2 ( A A 1 ) { k }_{ 2 }{ A }_{ 2 }={ k }_{ 2 }{ (A- }{ A }_{ 1 }) , because springs are connected consistently.

k 1 A 1 = k 2 ( A A 1 ) ( k 1 + k 2 ) A 1 = k 2 A A 1 = k 2 ( k 1 + k 2 ) A { k }_{ 1 }{ A }_{ 1 }={ k }_{ 2 }{ (A- }{ A }_{ 1 })\\ ({ k }_{ 1 }+{ k }_{ 2 }){ A }_{ 1 }={ k }_{ 2 }A\\ { A }_{ 1 }=\frac { { k }_{ 2 } }{ ({ k }_{ 1 }+{ k }_{ 2 }) } A

How do the springs know the mass is oscillating and the wall is still?

Guy Benians - 3 years, 10 months ago

Log in to reply

The spring isn't sentient, it doesn't know :) Even if you assume the mass is still and the wall is oscillating, you get the same results for this problem. Try it out!

Pranshu Gaba - 3 years, 10 months ago

Log in to reply

Thanks. I initially thought it should be symmetrical.

Guy Benians - 3 years, 10 months ago

It seems to me that the claim that the forces must be equal is different from the claim that the spring is connected; if there was no net force at point P, then P would not accelerate at all.

Peter Boyajian - 3 years, 10 months ago

Log in to reply

At time t=0, this is not a problem, however. My bad

Peter Boyajian - 3 years, 10 months ago

Log in to reply

The point P has no mass, so can we can say the net force (mass × \times acceleration) on it is zero for all t t ?

Pranshu Gaba - 3 years, 10 months ago

Log in to reply

@Pranshu Gaba Changing the mass of P should not change the net force on P; it should only change the acceleration that P experiences as a result of the force. Furthermore, I believe that is would make more sense to consider an infinitesimal segment of the spring of length dl and mass dm; this seems much more physical than considering a point.

Peter Boyajian - 3 years, 10 months ago

@Peter Boyajian just because it is at t=0 does not mean that it shouldn't accelerate. I believe the reason that k1x1= k2x2 because all internal forces must balance for an object to be "continuous"

William G. - 3 years, 10 months ago

Log in to reply

@William G. What exactly is your definition of continuous?

Peter Boyajian - 3 years, 10 months ago

Log in to reply

@Peter Boyajian that the spring does not fall apart at point p or any other point

William G. - 3 years, 10 months ago

If we go otherway round then we'll get the answer reversed?

Himanshu Yadav - 3 years, 10 months ago

Log in to reply

No

k 1 ( A A 2 ) = k 2 A 2 ( k 1 + k 2 ) A 2 = k 1 A A 2 = k 1 ( k 1 + k 2 ) A A 1 = A A 2 = k 2 ( k 1 + k 2 ) A { k }_{ 1 }(A-{ A }_{ 2 })=k_{ 2 }{ A }_{ 2 }\\ ({ k }_{ 1 }+k_{ 2 }){ A }_{ 2 }={ k }_{ 1 }A\\ { A }_{ 2 }=\frac { { k }_{ 1 } }{ ({ k }_{ 1 }+k_{ 2 }) } A\\ { A }_{ 1 }=A-{ A }_{ 2 }=\frac { { k }_{ 2 } }{ ({ k }_{ 1 }+k_{ 2 }) } A

Sardor Yakupov - 3 years, 10 months ago

I got the wrong answer k1/(k1+k2) because I mistook the spring constant for its stretchability, which is the reciprocal :-)

Michael Livshits - 3 years, 10 months ago
Peter Macgregor
Jul 31, 2017

I liked the Lagrangian magic of Steven Chase's solution. But suppose you were in an exam situation and didn't have time to do the detailed calculation?

I reasoned physically as follows. Suppose the two springs had the same spring constant. Then the point between them will oscillate with half the amplitude of the block. So we know the answer must be option four or five!

Now suppose we make k 2 > > k 1 k_2 >> k_1 so big that the spring attached to the block hardly stretches at all during the motion. Then P has almost as big an amplitude as the block, allowing us to identify the fifth option as the correct one.

... I used similar reasoning to get this wrong ... make the string attached to the brick be rigid (i.e. its string constant would be 0) and point P would oscillate with amplitude A. Similarly, if the string attached to the wall had 0 string constant then point P would not oscillate. So what I learned from your solution is that a rigid body (i.e. a body that doesn't stretch) has an infinite string constant. Knowing very little about string constants I assumed that a rigid string should have a zero spring constant (i.e. 0 springiness). ... sigh ...

David Hairston - 3 years, 10 months ago

Log in to reply

An easy way to see that a large k represents a stiff spring is to remember Hooke's law F = kx where F is the force and x is the extension of the spring. If k is very big then even a small x will produce a big force. Or to put it another way a big force will only stretch the spring a little!

Cheers!

Peter Macgregor - 3 years, 10 months ago

Suppose k2 is infinitely big, absolutely rigid spring; so, amplitude at P point is also "A". Only the last solution in possible.

José Ramón de Diego Luis - 3 years, 10 months ago

Peter i followed your reasoning at the fact of the two springs having the same spring constant, then i thought if the block M haves the A oscilation and it is directly attanched to the k2 then its more logical to be option 5

Relue Tamref - 3 years, 10 months ago
Steven Chase
Jul 20, 2017

This is a good opportunity to practice Lagrangian Mechanics . Define s 1 s_1 and s 2 s_2 as the stretches of the two springs.

Spring Potential Energy: U = 1 2 k 1 s 1 2 + 1 2 k 2 s 2 2 U = \frac{1}{2} k_1 s_1^2 + \frac{1}{2} k_2 s_2^2

Mass Kinetic Energy: E = 1 2 m ( s 1 ˙ + s 2 ˙ ) 2 E = \frac{1}{2} m (\dot{s_1} + \dot{s_2})^2

Lagrangian:

L = E U = 1 2 m ( s 1 ˙ + s 2 ˙ ) 2 1 2 k 1 s 1 2 1 2 k 2 s 2 2 L = E - U = \frac{1}{2} m (\dot{s_1} + \dot{s_2})^2 - \frac{1}{2} k_1 s_1^2 - \frac{1}{2} k_2 s_2^2

Equations of motion:

d d t L s 1 ˙ = L s 1 d d t L s 2 ˙ = L s 2 \frac{d}{dt} \frac{\partial{L}}{\partial{\dot{s_1}}} = \frac{\partial{L}}{\partial{s_1}} \\ \frac{d}{dt} \frac{\partial{L}}{\partial{\dot{s_2}}} = \frac{\partial{L}}{\partial{s_2}}

Evaluating these results in:

m ( s 1 ¨ + s 2 ¨ ) = k 1 s 1 m ( s 1 ¨ + s 2 ¨ ) = k 2 s 2 k 1 s 1 = k 2 s 2 m (\ddot{s_1} + \ddot{s_2}) = -k_1 s_1 \\ m (\ddot{s_1} + \ddot{s_2}) = -k_2 s_2 \\ \implies k_1 s_1 = k_2 s_2

Mass displacement (total stretch) in terms of spring stretches:

s = s 1 + s 2 = s 1 + k 1 k 2 s 1 = s 1 k 1 + k 2 k 2 s 1 = s k 2 k 1 + k 2 s = s_1 + s_2 = s_1 + \frac{k_1}{k_2} s_1 = s_1 \frac{k_1 + k_2}{k_2} \\ \implies \boxed{s_1 = s \frac{k_2}{k_1 + k_2}}

Very good derivation, which answers this question and prepares the way for many others about the two-spring system.

One minor detail: In the 'Equations of motion,' the derivatives on the right hand side (of L with respect to s1 and s2) should be partial derivatives. You used the full derivative vs partial derivative notation on the left hand sides, so I thought this was worth noting.

Matthew Feig - 3 years, 10 months ago

Log in to reply

Indeed, you're right. I have amended those equations. Thanks

Steven Chase - 3 years, 10 months ago

Splendid! good approach by using Lagrangian!

Kelvin Hong - 3 years, 10 months ago

Log in to reply

Thanks................................

Steven Chase - 3 years, 10 months ago

wait why does k1x1 have to equal k2x2? Is it because all internal forces must balance?

William G. - 3 years, 10 months ago

Log in to reply

There is no notion of force in my derivation. The equivalence comes right out of the derivation.

Steven Chase - 3 years, 10 months ago

Log in to reply

? But there has to be a reason why k1s1 = k2s2. Could you explain? Thanks

William G. - 3 years, 10 months ago

Log in to reply

@William G. Notice how in the two equations above that, the left sides are identical.

Steven Chase - 3 years, 10 months ago

Log in to reply

@Steven Chase @Steven Chase chase sir. Please suggest me good book for lagrangian from basic to high level .

A Former Brilliant Member - 1 year, 8 months ago

Log in to reply

@A Former Brilliant Member I didn't learn it from a book, but this wiki is a very good resource

https://brilliant.org/wiki/lagrangian-formulation-of-mechanics/

Steven Chase - 1 year, 8 months ago

Good solution

Sum Pan - 3 years, 10 months ago
Mo H
Aug 4, 2017

The force is always equal at two important points. One where the mass is attached to spring (of spring constant k2) and the point P acting on spring constant (k1). This is true since a force applied to spring with spring constant k2 will ultimately have to exert the same force on spring with spring constant k1 as they are always in tension. The maximum extension of spring with spring constant k1 = to the amplitude of point P. This maximum extension is created when the force acting on spring k1 with maximum force from hooke's law (F=Ke). We also know that the force on spring k1 is the same as on spring k2 hence their maximum extensions occur at the same time. The maximum extension of both springs added together is the extension of the spring. Hence from this information: F=k1 e1=k2 e2 e1+e2=A Given two different equations involving both e1 and e2 they can be simultaneously solved for e1 (the maximum extension of spring 1). Which is the amplitude of point P.

David Hiskiyahu
Aug 1, 2017

Assume spring K1 extremely hard and K2 extremely soft. K1 will not move almost, K2 will squeeze completely. This suggests the only possible solution - the last listed.

0 pending reports

×

Problem Loading...

Note Loading...

Set Loading...